You are on page 1of 51

DO NOT OPEN THIS TEST BOOKLET UNTIL YOU ARE TOLD TO DO SO

D
Scan for Synopsis PDF

TEST BOOKLET
GENERAL STUDIES
INDIAN ECONOMY-1
Time Allowed : Two Hours Maximum Marks :200
INSTRUCTIONS
1. IMMEDIATELYAFTER THE COMMENCEMENT OF THE EXAMINATION, YOU SHOULD CHECK THAT THIS TEST
BOOKLET DOES NOT HAVE ANY UNPRINTED OR TORN OR MISSING PAGES OR ITEMS, ETC. IF SO, GET IT
REPLACED BYA COMPLETE TEST BOOKLET.
2. Please note that it is the candidate's responsibility to encode and fill in the Roll Number and Test Booklet Series Code
A, B, C or D carefully and without any omission or discrepancy at the appropriate places in the OMR Answer Sheet.
Any omission/discrepancy will render the Answer Sheet liable for rejection.
3. You have to enter your Roll Number on the
Test Booklet in the Box provided alongside.
DO NOT write anything else on the Test Booklet.
4. This Test Booklet contains 100 items (questions). Each item is printed both in Hindi and English. Each item comprises
four responses (answers). You will select the response which you want to mark on the Answer Sheet. In case you feel
that there is more than one correct response, mark the response which you consider the best. In any case, choose ONLY
ONE response for each item.
5. You have to mark all your responses ONLY on the separate Answer Sheet provided. See directions in the Answer Sheet.
6. All items carry equal marks.
7. Before you proceed to mark in the Answer Sheet the response to various items in the Test Booklet, you have to fill in
some particulars in the Answer Sheet as per instructions sent to you with your Admission Certificate.
8. After you have completed filling in all your responses on the Answer Sheet and the examination has concluded, you
should hand over to the Invigilator only the Answer Sheet. You are permitted to take away with you the Test Booklet.
9. Sheets for rough work are appended in the Test Booklet at the end.
10. Penalty for wrong answers:
THERE WILL BE PENALTY FOR WRONG ANSWERS MARKED BY A CANDIDATE IN THE OBJECTIVE TYPE
QUESTION PAPERS.
(i) There are four alternatives for the answer to every question. For each question for which a wrong answer has
been given by the candidate, one-third of the marks assigned to that question will be deducted as penalty.
(ii) If a candidate gives more than one answer, it will be treated as a wrong answer even if one of the given answers
happens to be correct and there will be same penalty as above to that question.
(iii) If a question is left blank, i.e., no answer is given by the candidate, there will be no penalty for that question.

Join our Telegram channel for Synopsis PDF... https://t.me/IAS21stcenturyHYD)


income consisting of interest, rent, wage and
1) Which of the following are the issues related profits for his factor services. This is
to measurement of national Income through included in national income.
Income method in India? • Goods meant for Self-consumption: In
1. Wages and salary paid in Kind under-developed countries like India,
farmers keep a large portion of food and
2. Self-employed persons
other goods produced on the farm for self-
3. Goods meant for self-consumption consumption. The problem is whether that
Select the correct answer using the code part of the produce which is not sold in the
given below: market can be included in national income
or not. If the farmer were to sell his entire
(a) 1 only produce in the market, he will have to buy
(b) 1 and 2 only what he needs for self-consumption out of
his money income. If, instead he keeps some
(c) 2 and 3 only
produce for his self-consumption, it has
(d) 1, 2 and 3 money value which must be included in
Answer: D national income.

Explanation: • Wages and Salaries paid in Kind:


Another problem arises with regard to
The following problems arise in the computation wages and salaries paid in kind to the
of National Income by income method: employees in the form of free food, lodging,
• Owner-occupied Houses: A person who dress and other amenities. Payments in kind
rents a house to another earns rental by employers are included in national
income, but if he occupies the house income. This is because the employees
himself, will the services of the house-owner would have received money income equal
be included in national income. The services to the value of free food, lodging, etc. from
of the owner-occupied house are included the employer and spent the same in paying
in national income as if the owner sells to for food, lodging, etc.
himself as a tenant its services.
• Self-employed Persons: Another problem 2) Consider the following statements regarding
arises with regard to the income of self- Rebate of State and Central Taxes and Levies
employed persons. In their case, it is very (RoSCTL):
difficult to find out the different inputs
provided by the owner himself. He might 1) It was introduced by the Ministry of
be contributing his capital, land, labour and Finance.
his abilities in the business. But it is not 2) RoSCTL scrips cannot be used for special
possible to estimate the value of each factor Customs duties, cesses, and surcharges.
input to production. So he gets a mixed

KPIAS (2 - D) Cell: 91332 37733


3) It covers both State and Central levies. Development and Refinance Agency Bank.
Which of the statements given above are correct? 2) The fund will be used by public agencies to
(a) 1 and 2 only create urban infrastructure in tier-1 and
tier-2 cities.
(b) 2 and 3 only
Select the correct statement(s) from the code
(c) 1 and 3 only
given below:
(d) 1, 2 and 3
(a) 1 only
Solution (b)
(b) 2 only
# An initiative called the Rebate of State and
(c) Both 1 and 2
Central Taxes and Levies (RoSCTL) was
offered for embedded state and central (d) Neither 1 nor 2
duties and taxes that are not refunded Solution (d)
through/ Goods and Services Tax (GST). It
was/ introduced by the Ministry of Textiles. # UIDF will be established through the use
Hence, statement 1 is not correct. of priority sector lending shortfall. It will be
managed by the National Housing Bank. It
# The government delivers RoSCTL benefits will be established on the lines of the Rural
in the form of transferable duty credit Infrastructure Development Fund (RIDF).
scrips, not cash. These scrips can be used
only for payment of basic import duties. # The fund will be used by public agencies to
These cannot be used for special Customs create urban infrastructure in tier-2 and
duties, cesses, and surcharges. Hence, tier-3 cities. States will be encouraged to
statement 2 is correct. RoSCTL scrips are leverage resources from the grants of the
transferable. 15th Finance Commission, as well as
existing schemes, to adopt appropriate user
# RoSCTL covers both State and Central charges while accessing the UIDF.
levies. The entire RoSCTL burden,
however, ostensibly falls on the Central
Government as it issues and pays the 4) Consider the following statements regarding
RoSCTL scrips. Hence, statement 3 is Foreign Portfolio Investor (FPI):
correct. 1) They constitute a part of country’s capital
account are exhibited on its Balance of
3) With reference to ‘Urban Infrastructure Payments (BOP).
Development Fund (UIDF)’, consider the 2) FPIs don’t have any control over the
following statements: companies whose shares they hold.
1) UIDF will be established through the use 3) As compared to Foreign Direct Investment
of priority sector lending shortfall and will (FDI), FPU is more volatile and riskier.
be managed by the Micro Units
KPIAS (3 - D) Cell: 91332 37733
Which of the above statements are correct? Answer: B
(a) 1 and 2 only Explanation:
(b) 2 and 3 only * Corporate bonds are debt securities issued
(c) 1 and 3 only by private and public corporations.
(d) 1, 2 and 3 * Companies issue corporate bonds to raise
Solution (d) money for a variety of purposes, such as
building a new plant, purchasing equipment,
# FPIs are those that invest funds in markets or growing the business. At present, the
outside of their home turf. corporate bond market is regulated by the
Examples of FPIs include stocks, bonds, SECURITIES AND EXCHANGE
mutual funds, exchange traded funds, BOARD OF INDIA (SEBI) and the
American Depositary Receipts (ADRs), government bond market is regulated by the
and Global Depositary Receipts (GDRs). Reserve Bank of India (RBI). Hence,
OPTION B IS CORRECT.
* FPI is part of a country’s capital account
and is shown on its Balance of Payments 6. In the Product method or Gross Value Added
(BOP). Hence, statement 1 is correct. (GVA) method, the Gross Domestic Product
(GDP) is calculated by adding the gross value
# They are generally not active shareholders
added (GVA) of all firms in the economy.
and do not exert any control over the
Which of the following are used to find the
companies whose shares they hold. Hence,
GVA of a firm?
statement 2 is correct.
1. Sales of the firm
# FPI is often referred to as “hot money”
because of its tendency to flee at the first 2. Change in inventories
signs of trouble in an economy. FPI is more 3. Value of intermediate goods used
liquid, volatile and therefore riskier than
Foreign Direct Investment (FDI). Hence, Select the correct answer using the code
statement 3 is correct. given below.
(a) 1 and 3 only

5. The corporate bond market in India is (b) 1 and 2 only


regulated by: (c) 2 and 3 only
a) Reserve Bank of India (d) 1, 2 and 3
b) Securities and Exchange Board of India Answer: D
c) Insurance Regulatory and Development Explanation:
Authority of India
d) Department of Company Affairs
KPIAS (4 - D) Cell: 91332 37733
* Gross value added of firm = Gross value of
classified as either long- term or short-term
the output produced by the firm – Value of
capital movements. These transactions
intermediate goods used by the firm:
represent changes in a country’s ownership
* GVA = Value of sales by the firm + Value of assets and liabilities and can have
of change in inventories – Value of significant impacts on its financial stability
intermediate goods used by the firm and economic growth.
7) Which of the following is ‘not’ a component 8. With reference to Prompt Corrective Action
of Current Account Deficit? (PCA), which of the following statements is/
(a) Visible Trade are correct?

(b) Foreign Direct Investment (FDI) 1. It is a supervisory intervention of the RBI


to initiate and implement remedial measures
(c) Invisible Trade
for banks to restore its financial health.
(d) Transfer Payments
2. The prompt corrective action framework
Solution (b) is also imposed on cooperative banks and
# The current account of a country refers to non- banking financial companies.
its balance of short-term transactions, which Select the correct answer using the code
is determined by the disparity between its given below:
savings and investments. It encompasses
a) 1 only
various elements, including visible and
invisible trade, transfer payments, net factor b) 2 only
income, and remittances, all of which are c) Both 1 and 2
recognized as tangible transactions that can
d) Neither 1 nor 2
affect the economy’s income, output, and
employment levels by driving the movement Answer: A
of goods and services. * Explanation: RBI, under its supervisory
Foreign investment, such as FDI and framework, uses various measures/tools to
FPI, immovable properties, intangible maintain sound financial health of banks.
assets, trade credits, borrowings from other Under the Prompt Corrective Action (PCA)
nations, banking capital, and changes in the framework, RBI has specified certain
foreign exchange reserve are all regulatory trigger points in terms of three
components of the capital account. parameters, i.e. capital to risk weighted
assets ratio (CRAR), net non-performing
# The capital account of a country records
assets (NPA) and Return on Assets (RoA).
the flow of funds between the domestic
economy and the rest of the world. Its * The Reserve Bank of India (RBI) has
components include foreign investment, recently revised its prompt corrective action
loans, and capital transfers, which are (PCA) framework to exclude the

KPIAS (5 - D) Cell: 91332 37733


profitability parameter (Return on Asset) Answer: A
from its list of triggers.
Explanation:
* Capital, asset quality and leverage will be
the key areas for monitoring in the revised * Personal Income is the part of National
framework. Income which is received by households.
To calculate personal income we have to
* STATEMENT 1 IS CORRECT: The consider the following factors:
objective of the PCA Framework is to
enable supervisory intervention at an * Undistributed Profits: It is a part of the profit
appropriate time and require the Supervised which is earned by the firms and
Entity to initiate and implement remedial government enterprises and is not
measures in a timely manner, so as to restore distributed among the factors of production.
its financial health. We have to deduct UP from NI to arrive at
PI, since UP does not accrue to the
* STATEMENT 2 IS NOT CORRECT: The households. Increasing it will reduce
PCA Framework applies only to commercial personal income. Hence option 2 is not
banks and does NOT cover cooperative correct.
banks and non-banking financial companies.
RBI has brought in ‘Supervisory Action * Corporate taxes: Similarly, Corporate Tax,
Framework’ (SAF) for Urban Cooperative which is imposed on the earnings made by
Banks in place of PCA for commercial the firms, will also have to be deducted from
banks. the NI, since it does not accrue to the
households. Increasing it will reduce
personal income. Hence option 3 is not
9. Which of the following may lead to an correct.
increase in Personal Income in an economy?
* Net interest payments by the households:
1. Increase in national income The households do receive interest
payments from private firms or the
2. Increase in undistributed profits
government on past loans advanced by
3. Increase in corporate taxes them. And households may have to pay
Select the correct answer using the code interests to the firms and the government
given below. as well, in case they had borrowed money
from either. So, we have to deduct the net
(a) 1 only
interests paid by the households to the firms
(b) 1 and 2 only and government. Increasing it will reduce
(c) 2 and 3 only personal income.

(d) 1, 2 and 3 * Transfer payments: The households receive


transfer payments from government and
firms (pensions, scholarship, prizes, for

KPIAS (6 - D) Cell: 91332 37733


example) which have to be added to
calculate the Personal Income of the 1. It is a monetary policy tool which allows
households. Increasing it will increase the bank to borrow money through
personal income. repurchase agreements.
2. Under LAF, RBI performs short term and
Thus, Personal Income (PI) =
National Income- Undistributed profits – long term repo operations to inject liquidity
into the banking system.
Net interest payments made by households
– Corporate tax + Transfer payments to the Which of the statements given above is/are
households from the government and firms. correct?
Hence option 1 is correct. a) 1 only
b) 2 only
10) Windfall Tax sometimes seen in news refers c) Both 1 and 2
to
d) Neither 1 nor 2
(a) Tax levied on non-renewable energy
consuming industries Answer: C

(b) Tax on profit accrued from unprecedented Explanation:


events * Statement 1 is correct: Liquidity Adjustment
(c) Tax levied on import of luxury goods Facility (LAF) is a tool by which the
Reserve Bank adjust money supply in the
(d) Tax on companies earning profits from economy. It is a monetary policy tool which
foreign locations allows the bank to borrow money through
Solution (b) repurchase agreements (repo and reverse
* Windfall taxes are designed to tax the profits repos).
a company derives from an external, * Statement 2 is correct: Under Liquidity
sometimes unprecedented event— for Adjustment Facility (LAF), RBI does
instance, the energy price-rise as a result overnight as well as long term repo
of the Russia-Ukraine conflict. These are operations. The LAF works through various
profits that cannot be attributed to instruments devised by the RBI to inject
something the firm actively did, like an liquidity into the banking system when the
investment strategy or an expansion of system/institutions need cash as well as to
business. absorb liquidity when the banking system
has excess money.

11. With reference to Liquidity Adjustment 12. In the context of the Indian economy, which
Facility (LAF), consider the following of the following statements is/are correct
statements: about Wholesale Price Index (WPI)?
1. It measures the inflation in goods and

KPIAS (7 - D) Cell: 91332 37733


a) High government expenditure for poverty
services.
alleviation.
2. Manufacturing Products constitute the
b) Fiscal Responsibility and Budget
highest weightage in calculation of WPI.
Management Act, 2003
Select the correct answer using the code
c) Inflation targeting
given below:
d) Export promotional offers
a) 1 only
Answer: a
b) 2 only
* Macroeconomics is a branch of the
c) Both 1 and 2
economics field that studies how the
d) Neither 1 nor 2 aggregate economy behaves. It studies the
Answer: B behaviour and performance of an economy
as a whole.
Explanation:
* In macroeconomics, a variety of economy-
* The Wholesale Price Index is the price of a wide phenomena is thoroughly examined
representative basket of wholesale goods. WPI such as, inflation, price levels, rate of growth,
captures the average movement of wholesale national income, gross domestic product and
prices of goods and is primarily used as a GDP changes in unemployment.
deflator. WPI is released by the Economic Advisor
in the Ministry of Commerce and Industry. * Macro-economic stability means controlled
fiscal deficit, controlled current account
* Statement 1 is not correct: As services are deficit, and controlled inflation along with
not traded/transacted in the wholesale stable growth. High government
markets, WPI data does NOT include the expenditure for non-capital expenditure will
inflation due to services. result in possibility of de-stabilisation.
* Statement 2 is correct: The index basket of * The Fiscal Responsibility and Budget
the WPI covers commodities falling under Management Act, 2003 (FRBMA) is an Act
the three major groups namely Primary of the Parliament of India to institutionalize
Articles, Fuel and Power and Manufactured financial discipline, reduce India’s fiscal
products. deficit, improve macroeconomic
* MANUFACTURED PRODUCTS management and the overall management
CONSTITUTE THE HIGHEST of the public funds by moving towards a
WEIGHTAGE WITH 64.23 % followed by balanced budget.
Primary Articles with 22.62 % and Fuel and * Inflation targeting is a monetary policy
Power with 13.15 %. regime in which a central bank has an
explicit target inflation rate for the medium
term and announces this inflation target to
13. Which among the following is not a macro-
economic stabilisation policy?
KPIAS (8 - D) Cell: 91332 37733
the public. (a) 1 only
14. What do you understand by the term transfer (b) 2 only
payment?
(c) Both 1 and 2
(a) Payment given to the importers by a
(d) Neither 1 nor 2
domestic firm for the goods supplied.
(b) It is a part of profit which is not distributed
among the factors of production. Solution (c)
(c) It is a one-way payment to a person who # According to the Centre for Monitoring
has given or exchanged no money, good, or Indian Economy (CMIE), the labour force
service for it. consists of people who are 15 years or older,
and belong to either of the following two
(d) Payment made by the parent company to
categories: a) Are Employed b) Are
subsidiary companies for use of goods or
unemployed and are willing to work and are
Services.
actively looking for a job. These two
Answer: C categories have people “demanding” jobs.
* Explanation: A transfer payment is a one- This demand is what LFPR refers to.
way payment to a person who has given or Thus, the LFPR essentially is the
exchanged no money, good, or service for percentage of the working-age (15 years
it. It is a process used by governments as a or older) population that is asking for a job.
way to redistribute money through programs It represents the “demand” for jobs in an
such as old age or disability pensions, economy. It includes those who are
student grants, scholarships, prizes, employed and those who are unemployed.
unemployment compensation, etc.
Labour Force Participation as per Economic
Survey 2022-23:
`15) Which of the following statements are correct * Labour markets have recovered from the
regarding the Labour Force Participation effects of Covid-19, with unemployment
Rate as per Economic Survey 2022-23? rates falling from 5.8% in 2018-19 to 4.2%
1) Labour markets rebounded from Covid-19 in 2020-21. Hence, statement 1 is correct.
with a drop in unemployment rates in 2020- *- The Rural Female Labour Force
21 as compared to 2018-19. Participation Rate has risen from 19.7% in
2) An increase in Rural Female Labour Force 2018-19 to 27.7% in 2020-21, which is a
Participation Rate has been recorded in positive development. Hence, statement 2
2020-21 as compared to 2018-19. is correct. More than 75% of rural female
workers are employed in the agricultural
Select the correct answer using the codes
sector, implying a need to upskill and create
given below:

KPIAS (9 - D) Cell: 91332 37733


the firms
employment for women in this sector. Here,
self-help groups are serving as an effective 4. Personal tax and non-tax payments
conduit to tap the rising willingness of Select the correct answer using the code
females to work. given below.
(a) 1 and 4 only
16. Which one of the following statements best (b) 1, 3 and 4 only
describes the term ‘Dearness Allowance
(c) 2 and 3 only
(DA)’ given to government employees?
(d) 1, 2, 3 and 4
a) An allowance to meet a particular type of
expenditure over and above salary. Answer: D
b) It is a component of salary received Explanation:
towards the rent payment. * The part of National Income (NI) which is
c) It is a reward that is paid to an employee as received by households is referred to as the
an incentive for his work. Personal Income (PI).
d) It is paid by the government to its employees * Personal Income (PI) = NI – Undistributed
to offset the impact of inflation. profits – Net interest payments made by
households – Corporate tax + Transfer
Answer: D
payments to the households from the
Explanation: government and firms.
* ‘Dearness Allowance (DA)’ is provided by * However, even PI is not the income over
the government to its employees to cushion which the households have complete say.
the impact of the rising cost of living. They have to pay taxes from PI. If we
Inflation (or rate of increase in prices) eats deduct the Personal Tax Payments (income
away the buying power of money, hence tax, for example) and Non-tax Payments
the justification for DA. Dearness (such as fines) from PI, we obtain what is
Allowance is paid by the government to its known as the Personal Disposable Income.
employees as well as a pensioner to offset Thus Personal Disposable Income (PDI )
the impact of inflation. = PI – Personal tax payments – Non-tax
payments.

17. Which of the following needs to be subtracted


from the National Income to get the Personal 18. Consider the following statements:
Disposable Income of all households?
1. The government of India sets monetary
1. Net interest payments made by households policy for every five years.
2. Corporate Tax 2. The primary objective of monetary policy
3. Undistributed part of the profits earned by
KPIAS (10 - D) Cell: 91332 37733
is to maintain price stability. Select the correct answer using the code
Which of the statements given above is/are given below:
correct? (a) 1 only
a) 1 only (b) 2 only
b) 2 only (c) Both 1 and 2
c) Both 1 and 2 (d) Neither 1 nor 2
d) Neither 1 nor 2 Answer: D
Answer: B Explanation:
Explanation: * Factor cost includes only the payment to
* Statement 1 is not correct: Monetary Policy factors of production, it does not include any
refers to the policy of the central bank. In tax. In order to arrive at the market prices,
India, the Reserve Bank of India (RBI) is we have to add to the factor cost the total
responsible for monetary policy. Repo, indirect taxes less total subsidies.
Reverse Repo, CRR, SLR etc are part of * The basic prices lie in between: they include
monetary policy. the production taxes (less production
* Statement 2 is correct: The primary subsidies) but not product taxes (less
objective of monetary policy is to maintain product subsidies). Therefore in order to
price stability while keeping in mind the arrive at market prices, we have to add
objective of growth. Price stability is a product taxes (less product subsidies) to the
necessary precondition for sustainable basic prices, not the production taxes or
growth. To maintain price stability, inflation production subsidies.
needs to be controlled. The government of GVA at factor costs + Production taxes-
India sets an inflation target for every five Production subsidies = GVA at basic prices
years.
GVA at basic prices + Product taxes -
Product subsidies = GVA at market prices
19. With reference to methods of GDP * So, the interplay between product taxes and
estimation, consider the following statements product subsidies would determine if GVA
regarding ‘basic prices’: at Basic Prices would be less than or more
1. It includes the payment to factors of than GVA at market prices.
production but does not include any tax.
2. The Gross Value added at basic prices will 20. Consider the following statements regarding
always be less than Gross Value added at Reserve Bank of India (RBI):
market prices.
1. RBI acts as Manager of Foreign Exchange
KPIAS (11 - D) Cell: 91332 37733
under the Foreign Exchange Management * It acts as a banker to the government. It is
Act, 1999. the custodian of the foreign exchange
reserves of the economy and also acts as a
2. It acts as a banker to the government and bank to the banking system. Hence
bank of the banks. statement 2 is correct.
Which of the statements given above is/are
correct?
21) Which of the following is a part of Fiscal
a) 1 only Policy?
b) 2 only 1. Government expenditure
c) Both 1 and 2 2. Taxation by the Government
d) Neither 1 nor 2 3. Interest rate determination
Answer: C 4. Revenues from disinvestment of PSUs.
Explanation: Select the correct answer using the code
* The Reserve Bank of India was set up on given below:
the basis of the recommendations of the a) 1, 2, 3 and 4
Hilton Young Commission, under the
Reserve Bank of India Act, 1934. b) 1, 2 and 4 only

* RBI acts as the Manager of Foreign c) 1 and 2 only


Exchange of GoI under the Foreign d) 1, 2 and 3 only
Exchange Management Act, 1999.
Answer: b
* It facilitates external trade and payment and
* Fiscal policy is based on the theories of
promote orderly development and
British economist John Maynard Keynes.
maintenance of foreign exchange market
Also known as Keynesian economics, this
in India. Hence statement 1 is correct.
theory basically states that governments can
* It issues the currency of the country. This influence macroeconomic productivity levels
currency issued by the central bank can be by increasing or decreasing tax levels and
held by the public or by the commercial public spending.
banks and is called the ‘high-powered
* This influence, in turn, curbs inflation
money’ or ‘Reserve money’ or ‘monetary
(generally considered to be healthy when
base’ as it acts as a basis for credit creation.
between 2-3%), increases employment and
* It controls the money supply of the country maintains a healthy value of money. Fiscal
through various methods, like bank rates, policy is very important to the economy.
open market operations and variations in Proceeds from disinvestment are always
reserve ratios. taken into account while presenting the

KPIAS (12 - D) Cell: 91332 37733


statement 1 is not correct: Headline inflation
Union Budget.
measures price inflation arising due to all
* These receipts are treated as part of Capital types of commodities in the economy.
Receipts. Interest rate determination is the
* Core inflation measures the headline
job of Central bank and is a part of monetary
inflation EXCLUDING volatile components
policy.
i.e. food and fuel items.
* There are two types of fiscal policy. The
* statement 2 is correct: Core inflation is
first, and most widely-used, is expansionary.
considered a prime indicator of underlying
It stimulates economic growth. It’s most
long- term inflation. This is important
critical at the contraction phase of the
because it is used to calculate the impact
business cycle. The second type,
of rising prices on consumer income.
contractionary fiscal policy, is to stamp out
inflation.
23) Which of the following is a characteristic of a
Rentier Economy?
22. With reference to inflation, consider the
following statements: (a) High levels of inequality
1. Core inflation measures price inflation (b) A diversified economy with many sources
arising due to volatile components i.e. food of income
and fuel items. (c) Dependence on productive activity
2. Core inflation is considered a prime indicator (d) Low levels of unemployment
of underlying long-term inflation.
Solution (a)
Which of the statements given above is/are
correct? * A rentier economy is an economic system
where a significant portion of the income is
a) 1 only derived from rents, i.e., the payment made
b) 2 only for the use of assets such as land, natural
resources, patents, and intellectual property.
c) Both 1 and 2
* This results in a concentration of wealth
d) Neither 1 nor 2
among those who own these assets, leading
Answer: B to income inequality and dependence on
Explanation: passive income rather than productive
activity.
* Core inflation is an inflation measure which
excludes transitory or temporary price * Rentier economies are often characterized
volatility as in the case of some commodities by slow economic growth and high levels
such as food items, energy products etc. It of inequality.
reflects the inflation trend in an economy.

KPIAS (13 - D) Cell: 91332 37733


24) Consider the following statements 25. What are the reasons for not using GDP as
the indicator of well being of the people of a
1) Cess are resorted to only for specified Country?
purposes and it is discontinued when the
purpose for levying it is fulfilled. 1. It does not account for distribution of
income.
2) Cess is levied only on direct taxes and all
2. It takes into account only those economic
the cesses are utilized by the Central
Government. activities that are evaluated in monetary
terms.
Select the correct statement(s) using the
code given below: 3. It neglects negative externalities on health
and environment.
(a) 1 only
Select the correct answer using the code
(b) 2 only given below.
(c) Both 1 and 2 (a) 1 and 2 only
(d) Neither 1 nor 2 (b) 1 and 3 only
Solution (a) (c) 2 and 3 only
# They are resorted to only for a specific (d) 1, 2 and 3
purpose and are to be discontinued after
the objective is fulfilled. A cess is an Answer: D
additional tax levied by the government to Explanation:
raise funds for a specific purpose. Major Limitations of GDP
cesses include the health and education
cess, additional excise duties on petrol and There are several limitations of GDP as a welfare
diesel, road and infrastructure cess, national indicator.
calamity contingent duty, cesses on crude • GDP does not reflect inequality present
oil and exports. within the economy. Economic inequality is
# Cess can be levied both on direct and indirect not revealed by GDP figures.
taxes. Cesses and surcharges, are not part • Non-monetary exchanges: Many activities
of the divisible tax pool that is shared with in an economy are not evaluated in
states. Only GST compensation cess, which monetary terms. For example, domestic
are wholly appropriated by the states under services women perform at home are not
a revenue guarantee mechanism. paid for. This is a case of underestimation
of GDP. GDP ignores voluntary and
charitable work, social service as it is
unpaid.
• Externalities: Externalities refer to the
benefits (or harms) a firm or an individual
KPIAS (14 - D) Cell: 91332 37733
causes to another for which they are not Chairman of the MPC.
paid (or penalized). Externalities do not have Select the correct answer using the code
any market in which they can be bought given below:
and sold. For example, output of oil refinery a) 1 only
is taken into GDP calculation but pollution
and ill effects are not deducted from GDP. b) 2 only
This is the case of overestimation of GDP c) Both 1 and 2
d) Neither 1 nor 2
26) Consider the following statements: Answer: B
1. A higher GDP growth in an economy must Explanation:
increase employment growth.
* Statement 1 is not correct: The Monetary
2. A higher employment growth must decrease Policy Committee (MPC) constituted by the
unemployment rate. Central Government under Section 45ZB
Which of the statements given above is/are determines the policy interest rate required
correct? to achieve the inflation target. MPC has the
authority to decide only the repo rate and
a) 1 only not CRR & SLR.
b) 2 only * Statement 2 is correct: The committee will
c) Both 1 and 2 have six members. Of the six members, the
d) Neither 1 nor 2 government will nominate three. No
government official will be nominated to the
Answer: d MPC. The other three members would be
* Higher GDP growth may not always from the RBI with the governor being the
increase employment growth, a ex-officio chairperson. Deputy governor of
phenomenon called Jobless Growth. Even RBI in charge of the monetary policy will
higher employment growth may not always be a member, as also an executive director
decrease unemployment rate as it depends of the central bank.
also on how many people are entering the 28. For incom e accounting, the domestic
labour force (employed + unemployed). territory of a country includes which of the
following?
27. Which of the following statements is/are true 1. Territory within the political frontiers
regarding the Monetary Policy Committee 2. Oil and natural gas rigs operated by the
(MPC)? country in international waters
1. MPC has the authority to decide repo rate, 3. Embassies of other countries located on its
CRR & SLR. own territory
2. The Governor of RBI is ex-officio
KPIAS (15 - D) Cell: 91332 37733
Select the correct answer using the code Solution (d)
given below. # This report is the tenth edition of the
(a) 1 only (Organisation for Economic Co-operation
and Development) OECD’s Tax
(b) 1 and 2 only
Administration Series. It provides
(c) 2 and 3 only internationally comparative data on global
(d) 1 and 3 only trends in tax administrations across 58
advanced and emerging economies.
Answer: B
# The report is intended to inform and inspire
Explanation:
tax administrations as they consider their
The domestic territory of a country includes: future operations, as well as to provide
• Territory living within political frontiers, information on global tax administration
including its territorial waters. trends and performance for stakeholders
and policy makers.
• Ships and aircraft operated by the country
between political frontiers of two or more The report is structured around nine
countries. chapters that examine the performance of
tax administration systems, using an
• Oil and natural gas rigs, fishing vessels and extensive data set and a variety of examples
floating platforms operated by the country to highlight recent innovations and
in international waters. successful practices.
• Embassies, military establishments and # This edition also provides a first glimpse of
consulates of the country located in other the impact of the COVID-19 pandemic on
countries of the world.+ the work of tax administrations. The
* It does not include the embassies of other underlying data comes from the
countries and international organisations International Survey on Revenue
located on its own territory. Administration and the Inventory of Tax
Technology Initiatives.

29) Tax Administration 2022 report was released


by which of the following? 30) Consider the following statements with
respect to ‘Post Devolution Revenue Deficit
(a) United Nations Development Programme
Grant’.
(b) World Trade Organisation
1) It is provided to the States under Article
(c) International Monetary Fund 275 of the Constitution.
(d) Organisation for Economic Co-operation 2) It is released to the States as per the
and Development recommendations of the successive Finance
Commissions.
KPIAS (16 - D) Cell: 91332 37733
Select the correct statement(s) using the (c) 1 only
code given below: (d) 1, 2 and 3
(a) 1 only Solution (a)
(b) 2 only # Despite the downward revision, the growth
(c) Both 1 and 2 estimate for FY23 is higher than for almost
(d) Neither 1 nor 2 all major economies and even slightly above
the average growth of the Indian economy
Solution (c) in the decade leading up to the pandemic.
# The Post Devolution Revenue Deficit IMF estimates India to be one of the top
Grants are provided to the States under two fast-growing significant economies in
Article 275 of the Constitution. The grants 2022. Despite strong global headwinds and
are released to the States as per the tighter domestic monetary policy, if India is
recommendations of the successive Finance still expected to grow between 6.5 and
Commissions to meet the gap in Revenue 7.0%, and that too without the advantage
Accounts of the States post devolution. of a base effect, it is a reflection of India’s
underlying economic resilience. Hence,
# The eligibility of States to receive this grant
statement 3 is not correct.
and the quantum of grant for the period from
2020-21 to 2025-26 was decided by the # The rebound in consumption has also been
Fifteenth Finance Commission based on the supported by the release of “pent-up”
gap between assessment of revenue and demand, a phenomenon not again unique to
expenditure of the State after taking into India but nonetheless exhibiting a local
account the assessed devolution during this phenomenon influenced by a rise in the
period. share of consumption in disposable
income. Hence, statement 1 is correct.
# Capex thrust in the last two budgets of the
31) India is expected to grow between 6.5 and
Government of India was not an isolated
7.0% in FY23. Which of the following have
initiative meant only to address the
been India’s growth drivers in FY23
infrastructure gaps in the country. It was
according to Economic survey?
part of a strategic package aimed at
1) Rebound in domestic consumption. crowding-in private investment into an
2) Thrust on Public Capital expenditure. economic landscape broadened by the
vacation of non-strategic PSEs
3) It is due to advantage of ‘Base effect’. (disinvestment) and idling public sector
Select the correct answer using the code assets. Hence, statement 2 is correct.
given below:
(a) 1 and 2 only
(b) 2 and 3 only
KPIAS (17 - D) Cell: 91332 37733
32. With reference to Unified Logistics Interface * STATEMENT 2 IS NOT CORRECT:
Platform (ULIP), consider the following Unified Logistics Interface Platform
statements: (ULIP) was launched as part of the
1. It enables all logistics stakeholders to track ‘National Logistics Policy (NLP)’.
& trace consignment including
verification of drivers and vehicle details.
33. Which of the following statements is not
2. It was launched under the National correct about ‘Effective Revenue Deficit’?
Transport Policy. a) It excludes those grants given to States
Which of the statements given above is/are which are used by States for creation of
correct? capital assets.
a) 1 only b) Effective Revenue deficit will always be
b) 2 only lower than Revenue Deficit.

c) Both 1 and 2 c) It clearly distinguishes grants used for


consumption purposes and those
d) Neither 1 nor 2
or investments.
Answer: A
d) This concept was introduced in the year
Explanation: 2008-2009.
* Unified Logistics Interface Platform Answer: d
(ULIP) was recently launched by the
government as part of the ‘National * This concept was introduced in the Union
Budget for 2012-13 with the objective of
Logistics Policy (NLP)’, a promising
fiscal consolidation path to be adopted by
initiative in the logistics sector that aims to
bring ease of doing business in the logistics the government. Effective Revenue Deficit
sector by simplifying the logistics processes, will always be less than Revenue Deficit
improving its efficiency, bringing in as it excludes productive grants for capital
transparency and visibility, and reducing assets to States.
logistics cost & time. * Reason –While revenue deficit is the
* STATEMENT 1 IS CORRECT: ULIP will difference between revenue receipts and
give direct and indirect benefits to all revenue expenditure, the present
logistics stakeholders like verification of accounting system includes all grants from
drivers and vehicles details in a single click, the Union Government to the state
governments/Union territories/other bodies
tracking & tracing of consignment,
Route Optimization Planning, timely update as revenue expenditure, even if they are
on the destination of the consignment, used to create assets. Such assets created
reduce paper works, empty carrier & by the sub-national governments/bodies are
container visibility, inventory management owned by them and not by the Union
etc. Government. Nevertheless, they do result
KPIAS (18 - D) Cell: 91332 37733
35. It is calculated by adding residents’
in the creation of durable assets. Such investment incom e from overseas
revenue expenditures contribute to the investments and subtracting foreign
growth in the economy and therefore, should residents’ investment income earned within
not be treated as unproductive in nature. a country to the Gross Domestic Product
(GDP).
34) Consider the following statements: Which of the following concepts is explained
in the above paragraph?
1. Demographic Dividend is associated with
lower dependency ratio in the country. (a) Gross National Product
2. Such dividend is followed by Demographic (b) Net National Product
Echo under which old-age dependency ratio (c) Net Domestic Product
increases.
(d) Net National Income
Which of the statements given above is/are
correct? Answer: A

a) 1 only Explanation:

b) 2 only * Gross National Product (GNP) is an


estimate of the total value of all the final
c) Both 1 and 2 products and services turned out in a given
d) Neither 1 nor 2 period by the means of production owned
by a country’s residents. GNP is commonly
Answer: c
calculated by taking the sum of personal
* Demographic dividend occurs when the consumption expenditures, private domestic
proportion of working people in the total investment, government expenditure, net
population is high because this indicates that exports and any income earned by residents
more people have the potential to be from overseas investments, minus income
productive and contribute to growth of the earned within the domestic economy by
economy. foreign residents. Net exports represent the
* The dependency ratio is a measure showing difference between what a country exports
the number of dependents, aged zero to 14 minus any imports of goods and services.
and over the age of 65, to the total population, * GNP a” GDP + Factor income earned by
aged 15 to 64. It is also referred to as the 0the domestic factors of production
“total dependency ratio.” This indicator employed in the rest of the world – Factor
gives insight into the amount of people of income earned by the factors of production
nonworking age compared to the number of the rest of the world employed in the
of those of working age. domestic economy
* Hence, GNP a” GDP + Net factor income

KPIAS (19 - D) Cell: 91332 37733


Hence, OPTION D IS CORRECT.
from abroad

37. What is crowding out?


36. The State Bank of India has recently raised
the marginal cost of funds-based lending a) Decrease in private consumption due to rise
rates (MCLR). Which of the following factors in government expenditure
is/are considered for calculation of MCLR? b) Decrease in private investment due to rise
1. Marginal cost of deposits in government expenditure

2. Cost of maintaining CRR and SLR c) Decrease in private expenditure due to


inflation
3. Operational Costs of Banks
d) None of the above
Select the correct answer using the code
given below: Answer: b

a) 1 and 2 only * The crowding out effect is an economic


theory arguing that rising public sector
b) 2 and 3 only spending drives down or even eliminates
c) 1 and 3 only private sector spending. Sometimes,
government adopts an expansionary fiscal
d) 1, 2 and 3
policy stance and increases its spending to
Answer: D boost the economic activity.
Explanation: * Usually this increase in government
* From 1st April 2016, RBI introduced a new spending is funded using higher taxes or
methodology for calculation of the Base borrowing on part of the government.
Rates based on marginal cost of funds rather * When governments raise taxes in order to
than average cost of funds. This new introduce or expand welfare programs,
methodology is called Marginal Cost of individuals and businesses are left with less
Funds based Lending Rate (MCLR). discretionary income, which can reduce
* MCLR calculation methodology will be charitable contributions. In this respect,
based on the following factors: public sector expenditures for social welfare
can reduce private sector giving for social
- MARGINAL COST OF DEPOSITS/
welfare, offsetting the government’s
FUNDS
spending on those same causes.
- COST OF MAINTAINING CRR AND
SLR
38. With reference to Securities and Exchange
- OPERATIONAL COSTS OF BANKS
Board of India (SEBI), consider the following
- Tenor Premium (based on the time period statements:
for which loan is given)
1. It is a non-statutory body.
KPIAS (20 - D) Cell: 91332 37733
2. It is the regulator of capital markets in India. 4. Indian companies’ dependence on External
Which of the statements given above is/are Commercial Borrowings increases
correct? Select the correct answer using the code
a) 1 only given below:
b) 2 only a) 1 only
c) Both 1 and 2 b) 1 and 2 only
d) Neither 1 nor 2 c) 1, 2 and 3 only
Answer: B d) 1, 2, 3 and 4
Explanation: Answer: c
* The Securities and Exchange Board of India * Currency depreciation is a fall in the value
(SEBI)– Regulator of the financial markets of a currency in a floating exchange rate
in India was established on 12 th April 1988. system.
It was initially established as a non-statutory * Reasons – economic fundamentals, interest
body. rate differentials, political instability, risk
* STATEMENT 1 IS NOT CORRECT: aversion among investors. Currency
SEBI is a STATUTORY BODY in depreciation generally happens either due
accordance with the provisions of the to factors which increase demand for
Securities and Exchange Board of India Act, foreign currency like costlier imports or
1992. which decreases its supply like lower
exports or foreign investment outflow. ECBs
* STATEMENT 2 IS CORRECT: The
increase foreign currency supply.
primary objective of SEBI is to protect the
interest of people in the stock market and
provide a healthy environment for them. It 40. Who among the following can invest in the
regulates the capital market in India. Government Securities (G-Secs)?
1. Commercial banks
39) Indian currency can depreciate with respect 2. Foreign Portfolio Investors
to a foreign currency when
3. Individuals
1. The global price of imported crude oil
increases Select the correct answer using the code
given below:
2. Foreign investors take investments out of
India a) 1 and 2 only

3. Indian exports decline due to global b) 2 and 3 only


slowdown c) 1 and 3 only
KPIAS (21 - D) Cell: 91332 37733
d) 1, 2 and 3
3. Money
Answer: D
4. Capital
Explanation:
5. Entrepreneurship
* The RBI conducts auctions of G-secs
(Government-dated securities with original Select the correct answer using the code
maturity of one year or more) where given below.
institutional investors can place competitive (a) 1, 2 and 4 only
bids for them, and retail investors can apply (b) 1, 2, 3, 4 and 5
for allotment.
(c) 3 and 5 only
* OPTION 1 IS CORRECT: Auctions are
conducted on the electronic platform called (d) 1, 2, 4 and 5 only
the E-Kuber, the Core Banking Solution Answer: D
(CBS) platform of RBI. Commercial banks,
Explanation:
scheduled Urban Cooperative Banks
(UCBs), Primary Dealers (PD), insurance * The factor of production is “an economic
companies and provident funds are members term that describes the inputs that are used
of this platform. in the production of goods or services to
make an economic profit”.
* OPTION 2 IS CORRECT: Foreign Portfolio
Investors (FPIs) can also invest in • The factors of production include land, labor,
Government Securities (G-Secs) upto 6 % capital, and entrepreneurship. Out of these
of outstanding stocks of securities. four, entrepreneurship has recently been
accepted as a potential new factor of
* OPTION 3 IS CORRECT: The RBI has
production. Money is not a factor of
unveiled a scheme under which retail
production.
investors will be allowed to open retail direct
gilt accounts (RDG) directly with the central • Money is only a facilitator in the acquisition
bank. The retail direct scheme is a one-stop of those goods. Money facilitates trade and
solution to facilitate investment in stores value very effectively, individuals
government securities (G-secs) by individual cannot eat, wear or be sheltered by their
investors. bank account balances. The ultimate aim
of economic activity, work, and trade, is to
acquire goods, not money.
41. Which of the following are the “factors of
production” used in the production of goods
and services in economic activity?
1. Land
2. Labour

KPIAS (22 - D) Cell: 91332 37733


b) Statutory Liquidity Ratio
42. Consider the following statements:
c) Repo Rate
1. Nominal GDP is the value of GDP at the
d) Open Market Operations
current prevailing prices.
Answer: a
2. Real GDP is calculated in a way such that
the goods and services are evaluated at * Cash Reserve Ratio (CRR) – It is the
constant prices. percentage of cash deposits that banks need
to keep with the Reserve Bank of India on
Which of the statements given above are correct?
a fortnightly basis.
(a) 1 only
* Statutory Liquidity Ratio (SLR) – Apart
(b) 2 only from CRR, banks have to invest certain
(c) Both 1 and 2 percentage of their deposits in specified
financial securities like Central Government
(d) Neither 1 nor 2
or State Government securities. Unlike
Answer: C CRR, banks earn some amount on it.
Explanation: * Repo Rate and Reverse Repo Rate. Repo
* Statement 1 is correct: Nominal GDP, is the rate is the rate at which RBI lends to its
value of GDP at the current prevailing clients generally against government
prices. securities.

* Statement 2 is correct: GDP. Real GDP is * Open market operations (OMO) refer to
calculated in a way such that the goods and the buying and selling of government
services are evaluated at some constant set securities in the open market in order to
of prices (or constant prices). expand or contract the amount of money in
the banking system.
* Therefore, in order to compare the GDP
figures (and other macroeconomic
variables) of different countries or to 44. The Sensex index has been continuously
compare the GDP figures of the same rising in recent days. Which of the following
country at different points of time, we factors affect the change in market
cannot rely on GDPs evaluated at current movements?
market prices. For comparison, we take the
1. Rate of inflation in the economy
help of real GDP.
2. Extreme weather events
3. Level of trust placed on the legal system
43) Which of the following instruments of
Monetary Policy does not involve the use of Select the correct answer using the code
government securities? given below:

a) Cash Reserve Ratio a) 1 and 2 only

KPIAS (23 - D) Cell: 91332 37733


b) 2 and 3 only
c) 1 and 3 only 45. Gross National Product (GNP) is the GDP of
d) 1, 2 and 3 a country added with its ‘income from
abroad’. Which of the following is/are part of
income from abroad in India?
Answer: D 1. Private Remittances
Explanation: 2. Participation in international games by
* The stock market (Sensex and Nifty in Indian Sports persons
India) is a complex, interrelated system 3. Interest on External loans
composed of large and small investors
making uncoordinated decisions about a Select the correct answer using the codes
huge variety of investments. given below:

* OPTION 1 IS CORRECT: The law of (a) 1 only


supply and demand holds true as in any (B) 1 and 2 only
market. Some factors, such as the RATE (C) 1 and 3 only
OF INFLATION, have the power to move
the market as a whole higher or lower. (D) 1, 2 and 3

- Events that affect investor confidence and Answer: C


cause the market to move up and down Explanation:
include:
* Gross National Product (GNP) is the GDP
* OPTION 2 IS CORRECT: Natural of a country added with its ‘income from
disasters or extreme weather events abroad’. Here, the trans-boundary economic
* OPTION 3 IS CORRECT: Changes in the activities of an economy are also taken into
level of trust placed on the legal system account. The items which are counted in
the segment ‘Income from Abroad’ is:
- The publication of economic indicators such
as the Consumer Confidence Index • Private Remittances

- Wars or other conflicts • Interest on external loans

- Concerns over inflation or deflation • External grants

- Government fiscal and monetary policy


- Technological changes 46) What was the reason for the deviation of the
credit-to-GDP ratio from its trend during the
- Corporate or government performance data second decade of the millennium?
- Regulation or deregulation (a) A lack of investment opportunities in the
- Changes in the level of trust placed in an private non-financial sector.
industry such as the financial sector
KPIAS (24 - D) Cell: 91332 37733
(b) An increase in the number of bad loans in central government.
the banking sector. 2. These loans are included in the national
(c) A decrease in the demand for credit in the fiscal deficit.
private non-financial sector. 3. These borrowings can be used to fund both
(d) An improvement in the banking sector’s capital and revenue expenditure.
credit supply. Select the correct answer using the code
Solution (b) given below.
# Unsurprisingly, the credit to the private non- (a) 1 and 3 only
financial sector as a percent of GDP was (b) 2 only
consistently below its trend value for most
of the second decade of the millennium, (c) 3 only
implying a negative credit gap to GDP ratio. (d) 1, 2 and 3
# The gap worsened to an unprecedented Answer: a
level of 25% in 2017. A statistically
* Off-budget borrowings are loans that are
significant negative correlation (-0.5)
taken not by the Centre directly, but by
between the Gross NPAs and credit growth
another public institution that borrows on
shows that the banks’ credit supply was
the directions of the central government.
severely constrained due to stress in their
Such borrowings are used to fulfil the limits.
balance sheet during the second decade.
Both the centre and states are making such
That explains the large negative deviation
borrowings. Hence statement 1 is correct.
of the credit-to-GDP ratio from its trend.
* Off-budget borrowing figures did not include
the loans that public sector undertakings
were supposed to take on their behalf or
the deferred payments of bills and loans by
the Centre. Loans and deferred payments
are not part of the fiscal deficit calculation.
Hence statement 2 is not correct.
* Such borrowings are made by state-owned
firms to fund government schemes but are
not part of the official budget calculations.
47) With reference to off-budget borrowings by Other public sector undertakings have also
the Indian government, which of the following been borrowed from the government. For
statements is/are correct? instance, public sector oil marketing
companies were asked to pay for subsidized
1. These loans are not taken by the centre gas cylinders for Pradhan Mantri Ujjwala
directly, but by another public institution that Yojana beneficiaries in the past. Public
borrows on the directions of the
KPIAS (25 - D) Cell: 91332 37733
sector banks are also used to fund off- the old concept on the manufacture of goods
budget expenses. For example, loans from or on sale of goods or on provision of
PSU banks were used to make up for the services.
shortfall in the release of fertilizer subsidies. b) Destination based Taxation: GST is based
Hence statement 3 is correct.
on the principle of destination-based
48) Consider the following statements regarding consumption taxation as against the present
Goods and Services Tax (GST): principle of origin-based taxation.
1) It is value-added tax levied on most goods c) Dual GST: It is a dual GST with the Centre
and services sold for domestic consumption and the States simultaneously levying tax
within the country. on a common base. GST to be levied by
2) Import of goods or services are treated as the Centre is called Central GST (CGST)
inter-state supplies and are subject to and that to be levied by the States is called
Central GST (CGST). State GST (SGST). Import of goods or
services would be treated as inter-state
3) Central GST (CGST), State GST (SGST) supplies and would be subject to Integrated
and Integrated Goods & Services Tax Goods & Services Tax (IGST) in addition
(IGST) are levied at rates to be mutually to the applicable customs duties. Hence,
agreed upon by the Centre and the States. statement 2 is not correct.
Which of the statements given above is/are d) GST rates to be mutually decided: CGST,
correct? SGST & IGST are levied at rates to be
(a) 1 and 2 only mutually agreed upon by the Centre and the
States. The rates are notified on the
(b) 2 and 3 only
recommendation of the GST
(c) 1 and 3 only Council. Hence, statement 3 is correct.
(d) 1, 2 and 3 e) Multiple Rates: The schedule or list of items
Solution (c) that would fall under these multiple slabs
are worked out by the GST council.
# The Goods and Services Tax (GST) is a
value-added tax levied on most goods and
services sold for domestic consumption. The 49) An economy is going through a slowdown.
GST is paid by consumers, but it is remitted Which of the following are likely to be true?
to the government by the businesses selling
1. Level of Effective Demand is coming down.
the goods and services. Hence, statement
1 is correct. 2. Growth of Effective Demand is coming
down.
# Main Features of GST:
3. Inflation rate is going up.
a) Applicable On supply side: GST is applicable
on ‘supply’ of goods or services as against 4. Unemployment rate is going up.

KPIAS (26 - D) Cell: 91332 37733


Select the correct answer using the code Answer: B
given below: Explanation:
a) 1 and 2 only * Option (B) is correct: Market prices prices
b) 2 and 4 only are the prices as paid by the consumers
c) 2 and 3 only Market prices also include product taxes
and subsidies. The term factor cost refers
d) 1, 2 and 4 only to the prices of products as received by the
Answer: b producers. Thus, factor cost is equal to
market prices, minus net indirect taxes.
* Effective demand refers to the willingness
and ability of consumers to purchase goods * Net indirect taxes are calculated by
at different prices. It shows the amount of subtracting subsidies from indirect taxes.
goods that consumers are actually buying. Factor Cost = Market Price-Net Indirect
In Keynesian economics, effective demand taxes = Market Price - (Indirect taxes-
is the point of equilibrium where aggregate subsidies)
demand equals aggregate supply. * The market price is greater than factor cost
* ED is same as the National Income of the when the net indirect tax is positive, that is,
economy generally measured through GDP. indirect taxes are more than subsidies and
Slowdown only means decrease in GDP market prices are less than the factor cost
growth with its level still increasing. when the net indirect tax is negative, that
Normally inflation rate is lower during a is, indirect taxes are less than the subsidies
slowdown except when it is due to supply
side constraints. Unemployment rate
normally increases in any type of slowdown 51. Consider the following statements:
as with less GDP growth, job creation is 1. Repo Rate is the rate at which the central
affected. bank lends short term money to banks.
2. The Repo Rate is always higher than the
50. In which of the following cases will the factor Reverse Repo Rate.
cost will be greater than the market price of Which of the statements given above is/are
a product? correct?
(a) When subsidies are less than the indirect a) 1 only
taxes. b) 2 only
(b) When subsidies are more than indirect taxes. c) Both 1 and 2
(c) When the labour cost increase substantially. d) Neither 1 nor 2
(d) When the cost of marketing and Answer: C
advertisement decrease.
Explanation:
KPIAS (27 - D) Cell: 91332 37733
* Statement 1 is correct: Repo Rate, or virtually everyone and everything together
repurchase rate, is the key monetary policy including machines, objects, and devices.
rate of interest at which the central bank or * STATEMENT 1 IS CORRECT: 5G is based
the Reserve Bank of India (RBI) lends short on OFDM (Orthogonal frequency-division
term money to banks, essentially to control
multiplexing), a method of modulating a
credit availability, inflation, and the economic digital signal across several different
growth.
channels to reduce interference.
* Statement 2 is correct: Reverse Repo Rate * STATEMENT 2 IS CORRECT: 5G will
is the rate at which RBI borrows money bring wider bandwidths by expanding the
from the banks for the short term. The Repo usage of spectrum resources, from sub-3
Rate always stands higher than the Reverse GHz used in 4G to 100 GHz and beyond.
Repo Rate, and the spread between the two
is RBI’s income.
53. While calculating the Gross Domestic
Product (GDP) by expenditure method, which
52. With reference to 5th generation mobile of the following factors are taken into
network, consider the following statements: account?
1. 5G is based on a method of modulating a 1. Final household consumption expenditure
digital signal across several different
channels to reduce interference. 2. Expenditure on intermediate goods

2. 5G will bring wider bandwidths by 3. Final capital expenditure


expanding the usage of spectrum resources. 4. Government expenditure on unemployment
Which of the statements given above is/are allowance
correct? Select the correct answer using the code
a) 1 only given below.

b) 2 only (a) 1 and 4 only

c) Both 1 and 2 (b) 1, 3 and 4 only

d) Neither 1 nor 2 (c) 1 and 3 only


(d) 2, 3 and 4 only

Answer: C Answer: C

Explanation: Explanation:

* 5G is the 5th generation mobile network. It * The expenditure method is a system for
is a new global wireless standard after 1G, calculating gross domestic product (GDP)
2G, 3G, and 4G networks. 5G enables a new that combines consumption, investment,
kind of network that is designed to connect government spending, and net exports. It is

KPIAS (28 - D) Cell: 91332 37733


the most common way to estimate GDP. in total expenditure. The combination of the
* This final expenditure is made up of the sum two gives us Net Exports.
of 4 expenditure items, i.e GDP=
C+I+G+X-M namely: 54) Demand pull inflation in India can result from
* Hence option 1 is correct: Consumption (C): 1. Increase in proportion of young-age
Personal Consumption made by households, population in the country
the payment of which is paid by households
directly to the firms which produced the 2. Government reducing subsidy on electricity
goods and services desired by the bills
households. 3. Decrease in Indian exports
* Investment Expenditure (I): Investment is Select the correct answer using the code
an addition to the capital stock of an given below:
economy in a given time period. It includes
a) 1 only
capital expenditures by firms on assets, such
as equipment, production facilities, and b) 1 and 2 only
plants. c) 1 and 3 only
* Option 2 is not correct and option 3 is d) 1, 2 and 3
correct: It is to be noted that final investment
includes investment on capital goods and Answer: a
not on intermediate goods. * Demand-pull inflation results from strong
* Government Expenditure (G): It represents consumer demand. Many individuals
expenditures by the government on defense purchasing the same good will cause the
and non-defense goods and services, such price to increase, and when such an event
as weaponry, health care, and education. happens to a whole economy for all types
of goods, it is called demand-pull inflation.
* Option 4 is not correct: Government
expenditure on pension schemes, * Demand-pull inflation is used by Keynesian
scholarships, unemployment allowances economics to describe what happens when
etc. are not included in this as all of them price levels rise because of an imbalance
come under transfer payments. in the aggregate supply and demand. When
the aggregate demand in an economy
* Net Exports (X-IM): Expenditure on strongly outweighs the aggregate supply,
foreign-made products (Imports) are prices go up.
expenditure that escapes the system, and
must be subtracted from total expenditures. * Generally, the reasons are higher
In turn, goods produced by domestic firms consumption, investment, government
which are demanded by foreign economies expenditure, exports etc. In Keynesian
involve expenditure by other economies on theory, an increase in employment leads to
our production (Exports), and are included an increase in aggregate demand. Due to

KPIAS (29 - D) Cell: 91332 37733


the increase of demand, firms hire more unemployment will be low because total
people to increase their output. The more economic output is being maximized.
people firms hire, the more employment * Frictional unemployment is unemployment
increases. Eventually, output by firms during the intermediate time period between
becomes so small that the prices of their
two jobs. It is mostly voluntary and
goods rise. temporary in nature.

55) Shwetha was working as a software engineer 56) The most likely consequence of a higher
with an IT company. She quits her job to minimum wage in an economy is
prepare for Civil Services Examination.
Shwetha is (a) An increase in demand for labour

a) Seasonally Unemployed (b) Reduction in unemployment rate

b) Structurally Unemployed (c) An increase in supply of labour

c) Cyclically Unemployed (d) Increase in GDP growth rate

d) Frictionally Unemployed Answer: c

Answer: d * A minimum wage is the lowest


remuneration that employers can legally
* Seasonal unemployment occurs when pay their workers. Option (c) is Correct
people are unemployed at particular times
of the year when demand for labour is lower * A higher minimum wage attracts more
than usual. seasonal-unemployment. For people into the workforce, thereby
example, in a Ski resort unemployment is increasing the labour supply. Options (a),
likely to be higher in the summer when there (b) and (d) are Incorrect
is no snow. * A higher minimum wage may make
* Structural unemployment refers to a manufacturing or construction less
mismatch between the jobs available and competitive. Thus, higher minimum wage
the skill levels of the unemployed. Unlike can possibly reduce the labour demand.
cyclical unemployment, it’s caused by * Such a reduction in labour demand can
forces other than the business cycle. It result in increasing unemployment rate.
occurs when an underlying shift in the GDP growth rate is influenced by a very
economy makes it difficult for some groups large number of factors like manufacturing,
to find jobs. international trade, inflation, agriculture etc.
* Cyclical unemployment is a factor of overall Minimum wages alone can’t influence GDP
unemployment that relates to the cyclical growth rate.
trends in growth and production that occur
within the business cycle. When business
cycles are at their peak, cyclical
KPIAS (30 - D) Cell: 91332 37733
57) Which of the following gives Global Human given below:
Capital Index to the countries of the world? (a) 1 only
(a) World Health Organisation (b) 2 onlys
(b) World Bank (c) Both 1 and 2
(c) World Economic Forum
(d) Neither 1 nor 2
(d) UN Human rights council Solution (d)
Answer: c # GST Sahay is a digital platform developed
* The Global Human Capital Index is by the Goods and Services Tax Network
published by the World Economic Forum. (GSTN) in India to provide assistance to
* The GHCI measures countries’ ability to taxpayers in resolving their queries and
nurture, develop and deploy talent for grievances related to the Goods and
economic growth against four key areas of Services Tax (GST). Hence, statement 1 is
human capital development: correct.

o Capacity (determined by past investment # The GSTN was established as a not-for-


in formal education) profit, public-private partnership
company, with the aim of providing a shared
o Deployment (accumulation of skills through IT infrastructure and services to Central
work) and State Governments, taxpayers, and
o Development (reskilling and continued other stakeholders in the GST ecosystem.
upskilling of existing workers) Hence, statement 2 is correct.
o Know-how (specialised skills-use at work).
Note: The Human Capital Index is an annual 59) Stressed assets’ is an important indicator of
measurement prepared by the World Bank the health of banking system. Which among
whereas Global Human Capital Index is the following constitute stressed assets?
published by the World Economic Forum. 1. Non-performing assets
2. Written off assets
58) Which of the following statements is/ 3. Restructured loans
are incorrect?
Choose the correct answer using the code
1) The GST Sahay platform was created by given below
the GSTN to assist taxpayers in addressing
their concerns and questions related to GST. (a) 1 only

2) The GSTN was established as a not-for- (b) 1 and 3 only


profit, public-private partnership company. (c) 3 only
Select the correct answer using the code (d) 1, 2 and 3
KPIAS (31 - D) Cell: 91332 37733
Answer: d * Option (c) is correct: National Disposable
* Stressed assets are an essential indicator Income is the maximum amount of goods
of used to gauge the health of the banking and services a country has which could be
system. used for the purpose of saving and
consumption.
* Non-performing assets - A non performing
* It is obtained by adding Net National
asset (NPA) is a loan or advance for which
the principal or interest payment remained Product at market prices with current
overdue for a period of 90 days. transfers from the rest of the world. It gives
an idea of what is the maximum amount of
* Written off assets are those assets for which goods and services the domestic economy
the bank or lender doesn’t account the has its disposal. This is the maximum
money borrower owes to it. The financial amount of goods and services a country has
statement of the bank will indicate that the which could be used for the purpose of
written off loans are compensated through saving and consumption.
some other way.
* Restructured loans are new loans that
replaces the outstanding balance on an older 61. With reference to RBI’s balance sheet, which
of the following constitutes RBI’s income?
loan, and is paid over a longer period,
usually with a lower installment amount. 1. Printing of currency notes.
Loans are commonly rescheduled to
2. Returns earned on its foreign currency
accommodate a borrower in financial assets.
difficulty and, thus, to avoid a default.
3. Commission earned on managing
Stressed assets = NPAs + Restructured government borrowings.
loans + Written off assets
Select the correct answer using the code
given below:
60. The maximum amount of income available a) 1 and 2 only
within a domestic economy for the
consumption of goods and savings is termed b) 2 and 3 only
as: c) 1 and 3 only
(a) Gross Domestic Product d) 1, 2 and 3
(b) Gross National Product Answer: B
(c) National Disposable Income Explanation:
(d) Gross Value Added * The balance sheet of the Reserve Bank
Answer: C plays a critical role in the functioning of the
country’s economy largely reflecting the
Explanation: activities carried out in pursuance of its

KPIAS (32 - D) Cell: 91332 37733


currency issue function as well as monetary spend money.
policy and reserve management objectives.
* Option 1 is not correct: RBI’s expenditure 63) Reserve Bank of India acts as ‘the lender of
constitutes Printing of Notes, Expenditure the last resort’. It implies which of the
on Remittance of Currency, Agency following?
Charges, Employee Cost etc.
1. Other banks retain their deposits with the
* Option 2 & 3 are correct: RBI’s income
RBI
include Returns earned on its foreign
currency assets, Interest on its holdings of 2. RBI has the authority to demonetizes any
local rupee-denominated government bonds value of currency
or securities, Management commission on 3. RBI extends liquidity to bank to prevent a
handling the borrowings of State possible failure of the bank.
governments and the Central government.
Select the correct answer using the codes
given below:
62) Which among the following best describes (a) 1 and 2 only
‘Vote-on-Account’?
(b) 2 and 3 only
(a) The estimates of expenditure from the
(c) 3 only
Consolidated Fund included in the Annual
Financial Statement (d) 1, 2 and 3
(b) Grant in advance pending completion of Answer: c
procedure for the voting of the Demands Statement 1 – RBI as ‘Banker to Banks’
(c) Amount required to meet the expenditure * In order to facilitate a smooth inter-bank
charged on the Consolidated Fund transfer of funds, or to make payments and
(d) Grant for meeting an unexpected demand to receive funds on their behalf, banks need
upon the resources of India a common banker. By providing the facility
of opening accounts for banks, the Reserve
Answer: b
Bank becomes this common banker, known
* Vote on account is the permission to as ‘Banker to Banks’ function.
withdraw money from the Consolidated
* As Banker to Banks, the Reserve Bank
Fund of India in that period, usually two
provides short-term loans and advances to
months. Vote on account is a formality and
select banks, when necessary, to facilitate
requires no debate. When elections are
lending to specific sectors and for specific
scheduled a few months into the new
purposes. These loans are provided against
financial year, the government seeks vote
promissory notes and other collateral given
on account for four months. Essentially,
by the banks.
vote on account is the interim permission
of the parliament to the government to * Statement 3 – RBI as ‘lender of the last
KPIAS (33 - D) Cell: 91332 37733
resort’ (b) 2 only
* As a Banker to Banks, the Reserve Bank (c) Both 1 and 2
also acts as the ‘lender of the last resort’. (d) Neither 1 nor 2
It can come to the rescue of a bank that is
solvent but faces temporary liquidity Solution (b)
problems by supplying it with much needed # The Tax-to-GDP ratio is a measure of a
liquidity when no one else is willing to nation’s tax revenue relative to the size of
extend credit to that bank. The Reserve its economy. Hence, statement 1 is correct.
Bank extends this facility to protect the This ratio is used with other metrics to
interest of the depositors of the bank and to determine how well a nation’s government
prevent possible failure of the bank, which directs its economic resources via taxation.
in turn may also affect other banks and
# Developed nations typically have higher
institutions and can have an adverse impact
tax-to-GDP ratios than developing
on financial stability and thus on the
nations. Hence, statement 2 is not correct.
economy.
Higher tax revenues mean a country is able
* Statement 2 – RBI - legal aspect of to spend more on improving infrastructure,
demonetisation. Section 26(2) of RBI Act health, and education—keys to the long-
stipulates that on recommendation of the term prospects for a country’s economy and
Central Board of Directors of RBI, the people. According to the World Bank, tax
Central Government may, by notification in revenues above 15% of a country’s gross
the Gazette of India, declare that, with effect domestic product (GDP) are a key
from such date as may be specified in the ingredient for economic growth and,
notification. ultimately, poverty reduction.

64) Consider the following statements: 65. Consider the following statements.
1) The Tax-to-GDP ratio is a metric that 1. A base year is used for comparison in the
compares a country’s tax income to its measure of economic indices like GDP or
overall economic output. CPI.
2) Developing nations tend to have a greater 2. A change in the base year is essential to
proportion of their GDP coming from tax track structural changes in an economy.
revenues when compared to developed
3. Currently, in India the base year for GDP
nations.
calculation is 2011-12 and for CPI
Which of the statements given above is/ calculation is 2010.
are not correct?
Which of the statements given above is/are
(a) 1 only correct?
(a) 1, 2 and 3
KPIAS (34 - D) Cell: 91332 37733
(b) 1 and 3 only * Liquidity trap is a situation in which
(c) 1 and 2 only prevailing interest rates are low and savings
rates are high, making monetary policy
(d) 1 only ineffective. In a liquidity trap, consumers
Answer: C choose to avoid bonds and keep their funds
in savings, because of the prevailing belief
Explanation:
that interest rates will soon rise. Because
* A base year is the first of a series of years bonds have an inverse relationship to
in an economic or financial index. It is interest rates, many consumers do not want
typically set to an arbitrary level of 100. New, to hold an asset with a price that is expected
up-to-date base years are periodically to decline.
introduced to keep data current in a
* Should the regulatory committee try to
particular index. Any year can serve as a
stimulate the economy by increasing the
base year, but analysts typically choose
money supply, there would be no effect on
recent years.
interest rates, as people do not need to be
• A base year is used for comparison in the encouraged to hold additional cash.
measure of a business activity or economic
* As part of the liquidity trap, consumers
index. Hence, statement 1 is correct.
continue to hold funds in standard deposit
• A revision in the base year is essential for accounts, such as savings and checking
better policymaking. It is meant to track accounts, instead of in other investment
structural changes in an economy and options, even when the central banking
improve or update macroeconomic system attempts to stimulate the economy
indicators that reflect the economic through the injection of additional funds.
performances of a country. Hence, These consumer actions, often spurred by
statement 2 is correct. the belief of a negative economic event on
• Currently, the base year for GDP calculation the horizon, causes monetary policy to be
is 2011-12 and the base year for CPI generally ineffective.
calculation is 2012. Hence, statement 3 is
not correct.
67) Consider the following statements regarding
economic survey 2022-23:
66) Liquidity trap’ is associated with 1) India meets 60% of its edible oils demand
(a) Low interest rates and low savings rates through imports and sunflower oil, constitute
up to 15% of our total edible oil imports.
(b) High interest rates and low savings rates
2) Quantity and value of edible oil is
(c) Low interest rates and high savings rates continuously increased in last 5 years.
(d) High interest rates and high savings rates Which of the statements given above is/are
Answer: c correct?
KPIAS (35 - D) Cell: 91332 37733
(a) 1 only (a) Difference in the rate of inflation in
(b) 2 only successive years

(c) Both 1 and 2 (b) Difference between current level of real


GDP and Potential GDP
(d) Neither 1 nor 2
(c) Difference between rate of Inflation and
Solution (a) rate of deflation
# India meets 60 per cent of its edible oils (d) High or low levels of inflation in a previous
demand through imports, making it month distorting headline inflation of the
vulnerable to international movements in most recent month
prices. For instance, sunflower oil, which
makes up 15 per cent of our total edible oil Answer: b
imports, is procured mainly from Ukraine * An inflationary gap is a macroeconomic
and Russia. Hence, statement 1 is correct. concept that describes the difference
# Out of 60% of the edible oils consumed in between the current level of real gross
the country through imports, palm oil domestic product (GDP) and the anticipated
constitutes around 54% of the total edible GDP that would be experienced when an
oil imported mainly from Indonesia and economy is at full employment, also referred
Malaysia, while Soyabean oil constitutes to as the potential GDP.
around 25% and is imported from Argentina * For the gap to be considered inflationary,
and Brazil and Sunflower oil constitutes 15% the current real GDP must be the higher of
and is imported mainly from Ukraine and the two metrics.
Russia. * The inflationary gap exists when the
demand for goods and services exceeds
production due to factors such as higher
levels of overall employment, increased
trade activities or increased government
expenditure. This can lead to the real GDP
exceeding the potential GDP, resulting in an
inflationary gap.

# Statement 2 is self-explanatory from 69) Consider the following statements about PM


image. Hence, statement 2 is not correct. YUVA (Young, Upcoming and Versatile
Authors) 2.0 Scheme.
1. The scheme functions under the aegis of
68) Which of the following statement best the Department of Higher Education, the
describes the term “Inflationary Gap”? Ministry of Education.

KPIAS (36 - D) Cell: 91332 37733


2. The National Book Trust is the implementing Choose the incorrect statements:
agency of the scheme. a) 1 only
3. The scheme aims to mentor and train young b) 2 only
and budding authors below the age of 30
years. c) Both 1 and 2

Chose the correct statements: d) Neither 1 nor 2

a) 1, 2 and 3 Answer: a

b) 1 and 2 only * The National Informatics Centre (NIC)


was established in 1976 and is located in
c) 2 and 3 only New Delhi. It comes under the Ministry of
d) 1 and 3 only Electronics and Information Technology
Answer: a (MeitY).

* The Ministry of Education, Department of * Its main objective is to provide technology-


Higher Education, launched YUVA 2.0 - driven solutions to the Central and State
Prime Minister’s Scheme for Mentoring Governments. It has developed digital
Young Authors. governance platforms including MyGov,
PM-Kisan, National Power Portal, and e-
* The National Book Trust (NBT), India, as Shram. It offers free services to state
the implementing agency under MoE will government entities and funded by the
ensure the phase wise execution of the Central budget.
scheme under well-defined stages of
mentorship.
71) ‘Pahari’, a community seen in news are native
* The scheme is an Author Mentorship
programme to train young and budding of
authors (below 30 years of age) in order to a) Uttarakhand
promote reading, writing and book culture b) Jammu and Kashmir
in the country, and project India and Indian
writings globally. c) Ladakh
d) Arunachal Pradesh

70) Consider the following statements about Answer: b


National Informatics Centre. * The Pahari people or Pahari-speaking
1. It was established in 1976 and works under people is a cover term for a number of
the aegis of the Ministry of Science and heterogeneous communities inhabiting the
technology. Indian union territory of Jammu and
Kashmir, Himachal Pradesh and parts of
2. It has developed digital governance
Azad Kashmir administered by Pakistan.
platforms like MyGov and PM-Kisan.

KPIAS (37 - D) Cell: 91332 37733


Centre granted ST status to Pahari Which of the following constitutes the
community in J&K. Saptarishi priorities?
72) With reference to ‘Light Combat Helicopter- 1) Inclusive Development
Prachanda’, consider the following 2) Infrastructure and Investment
statements.
3) Unleashing the Potential
1. The LCH is the first indigenous Multi-Role
Combat Helicopter designed and 4) Digital Growth
manufactured by DRDO 5) Financial Sector
2. It is capable of operating from high altitude 6) Youth Power
terrain and carrying out precision strike at
7) Reaching the Last Mile
high altitude targets.
Select the correct answer using the codes
Select the correct statement(s)
given below:
a) 1 only
(a) 1, 2, 3, 4, 5 and 6 only
b) 2 only
(b) 1, 2, 4, 5, 6 and 7 only
c) Both 1 and 2
(c) 1, 2, 3, 5, 6 and 7 only
d) Neither 1 nor 2
(d) 1, 2, 3, 4, 5, 6 and 7
Answer: b
Solution (c)
* The LCH is the first indigenous MultiRole
Combat Helicopter designed and
manufactured by HAL. It has potent ground
attack and aerial combat capability.
* The helicopter possesses modern stealth
characteristics, robust armour protection
and formidable night attack capability.
Onboard advanced navigation system, guns
tailored for close combat and potent air to
air missiles make the LCH especially suited
for the modern battlefield. Capable of
operating from high altitude terrain and
carrying out precision strike at high altitude
targets.

73) The term “Saptarishi” was recently


mentioned in the Union Budget 2023-24.

KPIAS (38 - D) Cell: 91332 37733


of the industry to the government
74) With reference to ‘OPEC+’, consider the 3. Its mandate is to establish and operate the
following statements. National Accreditation Structure (NAS)
1. The non-OPEC countries which export Choose the correct answer using the code
crude oil are termed as OPEC plus countries given below
2. It is an informal grouping of three countries a) 3 only
which consists of Mexico, Russia and b) 1 and 2 only
United Arab Emirates.
c) 2 and 3 only
Select the correct statement(s)
d) 1 only
a) 1 only
Answer: a
b) 2 only
* Quality Council of India as a non-profit
c) Both 1 and 2 autonomous society registered under
d) Neither 1 nor 2 Societies Registration Act XXI of 1860 to
Answer: a establish an accreditation structure in the
country and spread the quality movement
* The non-OPEC countries which export in India by undertaking a National Quality
crude oil are termed as OPEC plus countries. Campaign.
These countries also participate in the
organisation’s initiatives such as voluntary * The Chairman of QCI is appointed by the
supply cuts in order to further bind policy Prime Minister on recommendation of the
industry to the government.
objectives between OPEC and non-OPEC
members. * Its mandate is to establish and operate the
National Accreditation Structure (NAS) for
* OPEC+ includes Azerbaijan, Bahrain,
Brunei, Kazakhstan, Malaysia, Mexico, conformity assessment bodies and providing
Oman, Philippines, Russia, Sudan and South accreditation in the field of health, education
Sudan. United Arab Emirates is a member and quality promotion.
of OPEC.
76) The budget documents presented to the
75) Consider the following statements with Parliament does not comprise of which of the
following:
respect to ‘Quality Council of India’.
1. It was set up as a non-profit autonomous (a) Outcome Budget
society under The Companies Act of 1956 (b) Consumption Profile
2. The Chairman of QCI is appointed by the (c) Expenditure Profile
Minister of Commerce and Industry on (d) Budget at a Glance
recommendation
KPIAS (39 - D) Cell: 91332 37733
Solution (b) every fifth year or earlier, as the deemed
* The budget documents presented to the necessary by the President.
Parliament comprise of the following: # Finance Commission (Miscellaneous
* Budget Speech, Annual Financial Provisions) Act, 1951 was passed by the
Statement, Demands for Grants, Parliament to determine the requisite
qualifications for appointment as members
Appropriation Bill, Finance Bill, Expenditure
Budget, Receipts Budget, Expenditure of the Commission and the procedure of
Profile, Memorandum Explaining the their selection.
Provisions in the Finance Bill, Budget at a # Every member will be in office for the time
Glance, Outcome Budget. period as specified in the order of the
President and is eligible for reappointment.
The recommendations of the Finance
77) Consider the following statements with Commission are not binding on the
respect to ‘Finance Commission’ government.
1) The Finance Commission is constituted by
the President at the end of every fifth year
or earlier as deemed necessary. 78) ‘Trade and Development report 2022’ was
released by
2) Finance Commission (Miscellaneous
Provisions) Act provides the requisite a) World Trade Organization
qualifications for appointment as members b) International Monetary Fund
of the Commission. c) World Bank
3) The recommendations of the Finance d) United Nations Conference on Trade and
Commission are binding on the government Development
and the members are not eligible for re-
appointment. Answer: d

Choose the correct answer using the code * Trade and Development Report is an annual
given below report released by UNCTAD. It provides
comprehensive and authoritative analysis of
(a) 1 and 2 only economic trends and policy issues of
(b) 2 only international concern. It offers
(c) 1 and 3 only recommendations for building a global
economy that ensures better future for all
(d) 3 only people and the planet.
Solution (a)
# The Finance Commission is constituted by 79. Consider the following statements about
the President under article 280 of the Non-Banking Financial Companies (NBFCs):
Constitution. It is constituted at the end of

KPIAS (40 - D) Cell: 91332 37733


1. They can lend and make investments. 80) Consider the following statements:
2. Deposits in NBFCs are guaranteed by the 1) Regressive Tax is a tax that takes a larger
Deposit Insurance and Credit Guarantee percentage of income from high-income
Corporation. groups than from low-income groups.
Which of the statements given above is/are 2) Proportional Tax is a tax that takes the same
correct? percentage of income from all income
groups.
a) 1 only
b) 2 only 3) Progressive Tax is a tax that takes a larger
percentage of income from low-income
c) Both 1 and 2 groups than from high-income groups.
d) Neither 1 nor 2 Which of the above statements are correct?
Answer: A (a) 2 only
Explanation: (b) 1 and 3 only
* Non-Banking Financial Company (NBFC) (c) 2 and 3 only
is a company registered under the
Companies Act, 1956 engaged in the (d) All of the above
business of loans and advances, acquisition Solution (a)
of shares/ stocks/ bonds/ debentures/ # In simple terms, tax is the money paid by
securities issued by Government or local the taxpayers to the government. Tax is
authority or other marketable securities. compulsory payment and not voluntary
* STATEMENT 1 IS CORRECT: NBFCs payment or donation made by the
lend and make investments and hence their taxpayers. It is compulsory as it is extracted
activities are akin to that of banks; however by the government through legislation. If
there are a few differences as given below: taxpayer fails to pay the taxes or evade
- NBFC cannot accept demand taxes, it is punishable by law. Government
deposits; performs various functions – such as
maintenance of law and order/ defence,
- NBFCs do not form part of the undertakes welfare and developmental
payment and settlement system and activities, provides public goods and
cannot issue cheques drawn on itself. services.
* STATEMENT 2 IS NOT CORRECT: # Types of Taxes according to fairness:
Deposit insurance facility of Deposit Progressive Tax, Proportional Tax and
Insurance and Credit Guarantee Regressive Tax.
Corporation is not available to depositors
of NBFCs, unlike in case of banks. a) Progressive Tax: A tax that takes
a larger percentage of income from
high-income groups than from low-
KPIAS (41 - D) Cell: 91332 37733
income groups. For example – d) Neither 1 nor 2
Income Tax rate India. (Tax rate Answer: a
goes up as income level rises). Rich
people pay a larger portion of their * Rule 4(2) covers the “identification of the
income as tax in comparison to poor first originator of the information”. The
people. extent of the first originator is limited to
India— “Provided further that where the
b) Proportional Tax: A tax that takes first originator of any information on the
the same percentage of income from computer resource of an intermediary is
all income groups. For example, if the located outside the territory of India, the first
Government decides to have a single originator of that information within the
slab in Income tax rate (let us assume territory of India shall be deemed to be the
30%). Then the percentage of first originator of the information.”
income paid in taxes will be same for
the low-income group, middle-income * The 2021 rules have stemmed from section
group and high- income group. 87 of the Information Technology Act, 2000
and are a combination of the draft
c) Regressive Tax: A tax that takes Intermediaries Rules, 2018 and the OTT
a larger percentage of income from Regulation and Code of Ethics for Digital
low-income groups than from high- Media.
income groups. For example, an
indirect tax on Hair Oil (let us
suppose – 18%). Lower income 82) ‘Reverse Flipping’ a term found in the
group pay high proportion of their Economic Survey is used in the context of
income as tax while buying it than
(a) Companies relying on input tax credit for
higher income group.
clearing dues
(b) Companies showing profits in offshore
81) Consider the following statements with locations to evade taxation
respect to ‘Information Technology Rules,
(c) Companies misusing inverted duty structure
2021’.
(d) Companies shifting their domicile back to
1. The rules provide for the identification of
India
the first originator of the information
Solution (d)
2. The rules have stemmed from Section 69
of the Information Technology Act * According to the Economic Survey 2022-
23 , Indian start-ups are exploring ‘reverse
Select the correct statement(s)
flipping’, or shifting their domicile back to
a) 1 only India, with easy access to capital from
b) 2 only private equity and venture capital, changes
in rules regarding round-tripping, and the
c) Both 1 and 2
KPIAS (42 - D) Cell: 91332 37733
growing maturity of India’s capital markets. Answer: D
Explanation:
83) ‘Cook Islands’ seen in news recently is POLICY TOOLS TO CONTROL MONEY
located in SUPPLY:
a) Atlantic Ocean *% Reserve Bank is the only institution which
can issue currency. This role of RBI, that
b) Indian Ocean
of being ready to lend to banks at all times,
c) Pacific Ocean is another important function of the central
d) Southern Ocean bank, and due to this the central bank is
said to be the lender of last resort.
Answer: c
* The tools used by the Central bank to
* The Cook Islands is a nation in the South
control money supply can be quantitative
Pacific, with political links to New Zealand.
or qualitative.
Its 15 islands are scattered over a vast area.
The largest island, Rarotonga, is home to * Quantitative tools: control the extent of
rugged mountains and Avarua, the national money supply by changing the CRR, or bank
capital. rate or open market operations. Hence
statement 1 is not correct.
* The United States (U.S.) President Joseph
Biden has announced U.S recognition of the * Qualitative tools include persuasion by the
Cook Islands as a sovereign state. Central bank in order to make commercial
banks discourage or encourage lending
which is done through moral suasion, margin
84. Consider the following statements regarding requirement, etc. Hence statement 2 is not
tools used by the RBI to control money correct.
supply:
1. Quantitative tools control the extent of
85. In the context of Indian economy, what does
money supply by moral suasion, margin
‘Stagflation’ mean?
requirement, etc.
a) There is a high rate of inflation accompanied
2. Qualitative tools include changing the CRR,
by a high employment rate.
or bank rate or open market operations.
b) There is a decline in the price level of goods
Which of the statements given above is/are
and services due to increased productivity.
correct?
c) There is a stagnant economic growth with
a) 1 only
high unemployment rate and inflation.
b) 2 only
d) There is a low level of inflation due to falling
c) Both 1 and 2 demand.
d) Neither 1 nor 2
KPIAS (43 - D) Cell: 91332 37733
Answer: C 87. The ‘Urban Co-operative Banks’, in India are
* Explanation: Stagflation is an economic regulated by:
condition when stagnant economic growth, a) Security and Exchange Board of India
high unemployment, and high inflation b) Registrar of State societies
combine together. Basically, inflation plus
stagnant growth equals stagflation. c) Banks Board Bureau
Stagflation is the stagnation in the economy d) Reserve Bank of India
accompanied by high inflation (instead of
Answer: D
low inflation due to falling demand).
Triggered first in 1973 by the OPEC’s * Explanation: Co-operative banks are
fourfold increase in oil prices which raised financial entities established on a co-
all prices, thus slowing down economic operative basis and belonging to their
growth. members. Reserve Bank of India is vested
with the responsibility of regulating and
supervising primary (urban) cooperative
86. In the context of Indian economy, which one banks, which are popularly known as Urban
of the following statements describes Cooperative Banks (UCBs). Hence,
‘Interest rate transmission’? OPTION D IS CORRECT.
a) Decrease in interest rate by RBI increases
the cost of loanable funds.
88. The Insurance Regulatory and Development
b) Increase in interest rate by RBI raises Authority of India (IRDAI) is the apex body
consumption demand. that supervises and regulates the insurance
c) Increase in interest rate by RBI raises sector in India. Which of the following is/are
investment. the functions of IRDAI?

d) Decrease in interest rate by RBI decreases 1. To safeguard the policyholder’s interest.


the cost of loanable funds. 2. To ensure faster and a hassle-free
Answer: D settlement of genuine insurance claims.

* Explanation: The interest rate transmission 3. To oversee the conduct of insurance


emerges as the dominant transmission companies in the financial markets.
mechanism of monetary policy. An Select the correct answer using the code
expansionary monetary policy (decrease in given below:
interest rate), for instance, is expected to
a) 1 and 2 only
lead to a lowering of the cost of loanable
funds, which, in turn, raises investment and b) 2 and 3 only
consumption demand and should eventually c) 1 and 3 only
be reflected in aggregate output and prices.
d) 1, 2 and 3

KPIAS (44 - D) Cell: 91332 37733


Answer: D statements:
Explanation: 1. It is the legal tender issued by a central
* The Insurance Regulatory and Development bank in a digital form.
Authority of India is a statutory regulatory 2. It is exchangeable one-to-one with the fiat
body set up in 1999 under the jurisdiction of currency.
the Ministry of Finance and is tasked with Which of the statements given above is/are
regulating and licensing the insurance and
correct?
reinsurance industries in India.
a) 1 only
Some of the functions of IRDAI include:
b) 2 only
- To safeguard the policyholder’s interest
while ensuring a fair and just treatment. c) Both 1 and 2
Hence, STATEMENT 1 IS CORRECT. d) Neither 1 nor 2
* To ensure faster and hassle-free settlement Answer: C
of genuine insurance claims. Hence,
Explanation:
STATEMENT 2 IS CORRECT.
* The Reserve Bank of India (RBI) has
* To promote fairness, transparency and
recently said it may introduce digital rupee,
oversee the conduct of insurance
the Central Bank Digital Currency (CBDC),
companies in the financial markets. Hence,
in phases beginning with wholesale
STATEMENT 3 IS CORRECT.
businesses in the current financial year.
* To have a fair regulation of the insurance
* STATEMENTS 1 & 2 ARE CORRECT:
industry while ensuring financial soundness
CBDC is the legal tender issued by a central
of the applicable laws and regulations.
bank in a digital form. It is the same as a
* To ensure the dealings are carried on in a fiat currency and is exchangeable one-to-
fair, integral manner along with financial one with the fiat Currency.
soundness keeping in mind the competence
90. With reference to Telecom Technology
of the insurance company.
Development Fund (TTDF), consider the
* To address the grievances of the following statements:
policyholder through a proper channel.
1. It aims to fund Research and Development
* To avoid malpractices and prevent fraud. in rural-specific communication technology
* To form a reliable management system with applications.
high standards of financial stability. 2. It was launched by the Universal Service
Obligation Fund (USOF).

89. With reference to Central Bank Digital Which of the statements given above is/are
Currency (CBDC), consider the following correct?

KPIAS (45 - D) Cell: 91332 37733


a) 1 only a) Delicensing of textile industry
b) 2 only b) Deregulation of cement price
c) Both 1 and 2 c) Repealing Foreign Exchange Regulation
d) Neither 1 nor 2 Act (FERA) 1973
d) Dereservation of telecom
Answer: C
Explanation: Answer: c

* Telecom Technology Development Fund * Microeconomics is the study of individuals,


(TTDF) Scheme was recently launched by households and firms’ behavior in decision
the Universal Service Obligation Fund making and allocation of resources. It
(USOF), a body under the Department of generally applies to markets of goods and
Telecommunications. The Scheme aims to services and deals with individual and
promote technology ownership and economic issues.
indigenous manufacturing, create a culture * The Foreign Exchange Regulation Act
of technology co-innovation, reduce imports, (FERA) 1973, imposed strict regulations on
boost export opportunities and creation of transactions involving foreign exchange and
Intellectual Property. Under the scheme, controlled the import and export of
USOF is also targeting to develop standards currency. FERA was repealed by the
to meet countrywide requirements and government in 1999 and replaced by the
create the ecosystem for research, design, Foreign Exchange Management Act.
prototyping, use cases, pilots, and proof of (FEMA), which liberalized foreign
concept testing, among others. exchange controls and removed many
* STATEMENT 1 IS CORRECT: Telecom restrictions on foreign investment.
Technology Development Fund (TTDF) * Repealing FERA is not sector sensitive and
aims to fund R&D in rural-specific therefore it is a macro-economic policy
communication technology applications and reform. All others are targeting specific
form synergies among academia, start-ups, sectors and so micro economic.
research institutes, and the industry to build
and develop the telecom ecosystem.
92) Consider the following statements:
* STATEMENT 2 IS CORRECT: Telecom
Technology Development Fund (TTDF) 1) The Revenue Deficit includes such
was launched by Universal Service transactions that affect the current income
Obligation Fund (USOF), a body under the and expenditure of the government.
Department of Telecommunications. 2) Fiscal Deficit equals the money the
government needs to borrow during the
year.
91. Which among the following is not a micro-
economic structural reform? 3) Primary Deficit equals Fiscal deficit minus
KPIAS (46 - D) Cell: 91332 37733
interest payments. Borrowing from RBI + Borrowing from
Which of the above statements are correct? abroad. The gross fiscal deficit is a key
variable in judging the financial health of
(a) 1 and 2 only the public sector and the stability of the
(b) 1 and 3 only economy.
(c) 2 and 3 only 3) Primary Deficit: Primary deficit equals
fiscal deficit minus interest payments. This
(d) All of the above
indicates the gap between the government’s
Solution (d) expenditure requirements and its receipts,
Measures of Government Deficit: not taking into account the expenditure
incurred on interest payments on loans taken
1) Revenue Deficit: It refers to the excess of during the previous years.
government’s revenue expenditure over
revenue receipts. Primary deficit = Fiscal deficit – Interest
payments
Revenue Deficit = Revenue expenditure –
Revenue receipts.
The revenue Deficit includes such 93) Which of the following statements is/are
transactions that affect the current income correct regarding Fiscal Responsibility and
and expenditure of the government. When Budget Management (FRBM) Act, 2003?
the government incurs a revenue deficit, it 1) It limits the annual budget deficit of the
implies that the government is dissaving and Central government to 3% of Gross
is using up the savings of the other sectors Domestic Product (GDP).
of the economy to finance a part of its
2) It ensures inter-generational equity in fiscal
consumption expenditure.
management and long-term macro-
2) Fiscal Deficit: It is the gap between the economic stability.
government’s expenditure requirements
Select the correct answer using the codes
and its receipts. This equals the money the
given below:
government needs to borrow during the
year. A surplus arises if receipts are more (a) 1 only
than expenditure. (b) 2 only
Fiscal Deficit = Total expenditure – (c) Both 1 and 2
(Revenue receipts + non-debt creating
capital receipts). It indicates the total (d) Neither 1 nor 2
borrowing requirements of the government Solution (c)
from all sources. * Fiscal Responsibility and Budget
From the financing side: Gross fiscal Management (FRBM) Act, 2003 aims to
deficit = Net borrowing at home + make the Central government responsible

KPIAS (47 - D) Cell: 91332 37733


for ensuring inter-generational equity in not have a direct impact on the assets and
fiscal management and long-term macro- liabilities of the government.
economic stability. Hence, statement 2 is 2) Salaries, Pensions, etc are the examples of
correct. Revenue Expenditure.
* The Act envisages the setting of limits on 3) Capital expenditure is used to create assets
the Central government’s debt and deficits.
or to reduce liabilities.
It limited the fiscal deficit to 3% of the GDP.
Hence, statement 1 is correct. Which of the above statements are correct?

* The NK Singh committee (set up in 2016) (a) 1 and 2 only


recommended that the government should (b) 1 and 3 only
target a fiscal deficit of 3% of the GDP in
(c) 2 and 3 only
years up to March 31, 2020 cut it to 2.8%
in 2020-21 and to 2.5% by 2023. (d) All of the above
Solution (d)
94) Consider the following statements with Components of Government Budget:
respect to ‘Union Budget 2022-23’ I) Revenue Budget: It consists of the Revenue
1) Goods and Services Tax forms the largest Expenditure and Revenue Receipts.
share of central revenue. * Revenue Receipts are receipts which do
2) Securities against small savings has the not have a direct impact on the assets and
highest share in financing deficits. liabilities of the government. It consists of
the money earned by the government
3) Effective Revenue Deficit as a percentage
through tax (such as excise duty, income
of GDP has consistently decreased over the
tax) and non-tax sources (such as dividend
last three years.
income, profits, interest receipts).
Choose the correct answer using the code
* Revenue Expenditure is the expenditure by
given below
the government which does not impact its
a) 1, 2 and 3 assets or liabilities. For example, this
b) 1 and 2 only includes salaries, interest payments, pension,
and administrative expenses.
c) 2 only
II) Capital Budget: It includes the Capital
d) 1 and 3 only
Receipts and Capital Expenditure.
Solution (c) * Capital Receipts indicate the receipts which
lead to a decrease in assets or an increase
95) Consider the following statements? in liabilities of the government. It consists
of: (i) the money earned by selling assets
1) Revenue Receipts are receipts which do (or disinvestment) such as shares of public

KPIAS (48 - D) Cell: 91332 37733


enterprises, and (ii) the money received in measured at current prices). Real GDP is
the form of borrowings or repayment of nominal GDP, adjusted for inflation to reflect
loans by states. changes in real output (It is the GDP
* Capital expenditure is used to create assets measured at constant prices).
or to reduce liabilities. It consists of: (i) the GDP Deflator = (Nominal GDP/ Real GDP)
long-term investments by the government x 100
on creating assets such as roads and Statement 2 is correct: The weights of
hospitals, and (ii) the money given by the goods and services in GDP deflator are not
government in the form of loans to states constant and differ according to the
or repayment of its borrowings. production level of each good and services
in the country. Due to these changes in
96. With reference to GDP deflator, consider the consumption patterns or the introduction of
following statements: new goods and services or structural
transformation are automatically reflected
1. It is the ratio of GDP measured at current in the deflator which is not the case with
prices to GDP measured at constant prices. other inflation measures.
2. It automatically reflects the changes in
consumption pattern and structural
transformations in the economy. 97. The term ‘Tax Expenditure’ implies:

Which of the statements given above is/are 1. Expenditure incurred by the government in
collecting taxes.
correct?
(a) 1 only 2. Total revenue from taxation as percentage
of total expenditure of the government.
(b) 2 only
3. Revenue foregone by the government due
(c) Both 1 and 2 to various exemptions and rebates given on
(d) Neither 1 nor 2 direct and indirect taxes.
Answer: C Which of the statements given above is/are
correct?
Explanation:
a) 1 and 3 only
* Statement 1 is correct: The Gross
Domestic Product (GDP) deflator is a b) 1 only
measure of general price inflation. It is c) 3 only
calculated by dividing nominal GDP by real
GDP and then multiplying by 100. Nominal d) 1, 2 and 3
GDP is the market value of goods and Answer: c
services produced in an economy, * Tax Expenditure corresponds to relaxations
unadjusted for inflation (It is the GDP given when tax burden becomes difficult

KPIAS (49 - D) Cell: 91332 37733


for the sustainability of a particular sector. Explanation:
Tax exemptions or incentives are given in * Option (d) is correct: Transfer expenditure
the form of lower rates of tax relative to relates to the expenditure in the form of
normal rates. Tax expenditures are revenue payments against which there is no
losses attributable to tax provisions that
corresponding return. Such expenditure
often result from the use of the tax system includes public expenditure on National Old
to promote social goals without incurring
Age Pension Schemes, Interest payments,
direct expenditures. Subsidies, Unemployment allowances,
* It does not relate to the expenditures Welfare benefits to weaker sections, etc.
incurred by the Government in the collection
of taxes. Rather it refers to the opportunity
cost of taxing at concessional rates, or the 99. Which of the following represents National
opportunity cost of giving exemptions, Income?
deductions, rebates, deferrals credits etc. (a) Net National Product at factor cost
to the tax payers. Tax expenditures indicate
(b) Net National Product at market prices
how much more revenue could have been
collected by the Government if not for such (c) Gross National Product at factor cost
measures. In other words, it shows the (d) Gross National Product at market prices
extent of indirect subsidy enjoyed by the
tax payers in the country. Answer: A
Explanation:

98. Expenditures on which of the following are * Option (a) is correct: Net National Product
considered Transfer expenditure of the at factor cost is also called as National
Government? Income.

1. Pension Schemes * Thus, NNP at factor cost a” National


Income (NI ) a” NNP at market prices –
2. Subsidies (Indirect taxes –Subsidies) a” NNP at
3. Unemployment allowances market prices – Net indirect taxes (Net
indirect taxes a” Indirect taxes – Subsidies)
Select the correct answer using the code
given below.
(a) 1 and 2 only 100) Consider the following statements with
respect to ‘Swachh Survekshan Survey 2022’.
(b) 1 and 3 only
1. Service level progress has the highest
(c) 2 and 3 only
weightage in determining the rank
(d) 1, 2 and 3
2. The survey was launched by the Ministry
Answer: D of Housing and Urban Affairs

KPIAS (50 - D) Cell: 91332 37733


3. Indore emerged as India’s first 7-star
Garbage Free city
Choose the correct answer using the code
given below
a) 1 and 2 only
b) 2 and 3 only
c) 1 and 3 only
d) All the above
Answer: d
* Service level progress has the highest
weightage in determining the rank with 40%
weightage. Citizens’ voice and certification
have 30% weightage each.
* Minister of Housing and Urban Affairs
(MoHUA) had launched the seventh
consecutive edition of Swachh Survekshan
(SS), the world’s largest urban cleanliness
survey conducted by Swachh Bharat
Mission-Urban (SBM-U). Designed with
‘People First’ as its driving philosophy,
Swachh Survekshan 2022 is curated
towards capturing the initiatives of cities for
the overall welfare and well-being of
frontline sanitation workers.
* Indore, the city of lakes and palaces, walked
away with the Cleanest City title for the
sixth consecutive year, in the ‘more than 1
lakh population’ category, while Surat was
adjudged the second cleanest city, for the
second time in a row.

KPIAS (51 - D) Cell: 91332 37733

You might also like